www.vorhilfe.de
- Förderverein -
Der Förderverein.

Gemeinnütziger Verein zur Finanzierung des Projekts Vorhilfe.de.
Hallo Gast!einloggen | registrieren ]
Startseite · Mitglieder · Impressum
Forenbaum
^ Forenbaum
Status VH e.V.
  Status Vereinsforum

Gezeigt werden alle Foren bis zur Tiefe 2

Navigation
 Startseite...
 Suchen
 Impressum
Das Projekt
Server und Internetanbindung werden durch Spenden finanziert.
Organisiert wird das Projekt von unserem Koordinatorenteam.
Hunderte Mitglieder helfen ehrenamtlich in unseren moderierten Foren.
Anbieter der Seite ist der gemeinnützige Verein "Vorhilfe.de e.V.".
Partnerseiten
Weitere Fächer:

Open Source FunktionenplotterFunkyPlot: Kostenloser und quelloffener Funktionenplotter für Linux und andere Betriebssysteme
Forum "Integration" - uneigentliche Integrale
uneigentliche Integrale < Integration < Funktionen < eindimensional < reell < Analysis < Hochschule < Mathe < Vorhilfe
Ansicht: [ geschachtelt ] | ^ Forum "Integration"  | ^^ Alle Foren  | ^ Forenbaum  | Materialien

uneigentliche Integrale: Tipp und Korrektur
Status: (Frage) beantwortet Status 
Datum: 20:01 Di 29.06.2010
Autor: SnafuBernd

Aufgabe
Berechnen Sie die folgenden uneigentlichen Integrale, sofern Sie existieren:
a) [mm] \integral_{0}^{\infty}{\frac{1}{1+e^x} dx} [/mm]
b) [mm] \integral_{0}^{1}{(ln(x))^2 dx} [/mm]
c) [mm] \integral_{1}^{e}{\frac{1}{x\sqrt{ln(x)}} dx} [/mm]

Hi,

zu a):
sei [mm] a\in ]0,\infty[ [/mm]
[mm] \integral_{0}^{a}{\frac{1}{1+e^x} dx} [/mm] = [mm] \integral_{0}^{a}{1-\frac{e^x}{1+e^x} dx} [/mm] , Subst.: [mm] u:=e^x [/mm] => [mm] dx=\frac{du}{u} [/mm]
[mm] =\integral_{0}^{a}{1 dx} [/mm] - [mm] \integral_{1}^{e^a}{\frac{u}{1+u} \frac{1}{u}du} [/mm]
=a- [mm] (ln(1+u)|^{e^a}_1 [/mm] = [mm] a-ln(1+e^a) [/mm] + ln(2)
[mm] wegen\limes_{a\rightarrow\infty}a-ln(1+e^a) [/mm] + ln(2) = [mm] \infty [/mm] - [mm] \infty [/mm] + ln(2)
Löschen sich die [mm] \infty [/mm] sich gegenseitig aus?
Habe ich irgendwo ein Fehler?

Snafu

        
Bezug
uneigentliche Integrale: Antwort
Status: (Antwort) fertig Status 
Datum: 20:13 Di 29.06.2010
Autor: rainerS

Hallo!

> Berechnen Sie die folgenden uneigentlichen Integrale,
> sofern Sie existieren:
>  a) [mm]\integral_{0}^{\infty}{\frac{1}{1+e^x} dx}[/mm]
>  b)
> [mm]\integral_{0}^{1}{(ln(x))^2 dx}[/mm]
>  c)
> [mm]\integral_{1}^{e}{\frac{1}{x\sqrt{ln(x)}} dx}[/mm]
>  Hi,
>  
> zu a):
>  sei [mm]a\in ]0,\infty[[/mm]
> [mm]\integral_{0}^{a}{\frac{1}{1+e^x} dx}[/mm] =
> [mm]\integral_{0}^{a}{1-\frac{e^x}{1+e^x} dx}[/mm] , Subst.: [mm]u:=e^x[/mm]
> => [mm]dx=\frac{du}{u}[/mm]
>  [mm]=\integral_{0}^{a}{1 dx}[/mm] -
> [mm]\integral_{1}^{e^a}{\frac{u}{1+u} \frac{1}{u}du}[/mm]
>  =a-
> [mm](ln(1+u)|^{e^a}_1[/mm] = [mm]a-ln(1+e^a)[/mm] + ln(2)
> [mm]wegen\limes_{a\rightarrow\infty}a-ln(1+e^a)[/mm] + ln(2) =
> [mm]\infty[/mm] - [mm]\infty[/mm] + ln(2)
>  Löschen sich die [mm]\infty[/mm] sich gegenseitig aus?

Ja. Tipp: schreibe [mm] $a=\ln e^a$: [/mm]

  [mm]a-\ln(1+e^a) = \ln e^a - \ln(1+e^a) = \ln \bruch{e^a}{1+e^a} = \ln \bruch{1}{e^{-a}+1} [/mm]

Viele Grüße
   Rainer


Bezug
        
Bezug
uneigentliche Integrale: b)
Status: (Frage) beantwortet Status 
Datum: 17:48 Mi 30.06.2010
Autor: SnafuBernd

Hi,

[mm] \integral_{0}^{1}{ln^2(x) dx} [/mm] , sei [mm] a\in[0,1] [/mm]
=> [mm] \integral_{a}^{1}{ln^2(x) dx} [/mm]  subst.: u:=ln(x) => [mm] dx=e^u [/mm] du
[mm] =\integral_{ln(a)}^{0}{u^2e^u du} [/mm]
[mm] =u^2e^u [/mm] - [mm] \integral_{ln(a)}^{0}{2ue^u du} [/mm]
[mm] =u^2e^u [/mm] - [mm] 2ue^u [/mm] + [mm] \integral_{ln(a)}^{0}{2e^u du} [/mm]
[mm] =u^2e^u|^0_{ln(a)} [/mm] - [mm] 2ue^u|^0_{ln(a)} +2e^u|^0_{ln(a)} [/mm]
=0 - [mm] ln^2(a)a [/mm] + 2ln(a)a + 2 - 2a
=a(2ln(a) - [mm] ln^2(a) [/mm] + [mm] \frac{2}{a} [/mm] -2)

ich habe mal im internet integriert und da kommt [mm] a(ln^2(a)-2ln(a)+2), [/mm] kann mir jemand sagen, wo mein Fehler liegt?

Snafu

Bezug
                
Bezug
uneigentliche Integrale: Antwort
Status: (Antwort) fertig Status 
Datum: 18:03 Mi 30.06.2010
Autor: schachuzipus

Hallo Snafu,

integriert hast du das Teil richtig.

Es hängt etwas an der unteren Grenze. Die obere ist mit $x=1$ richtig [mm] $u=\ln(x)=\ln(1)=0$ [/mm]

Die untere wird bei der Substitution "uneigentlich".

Nimm dir als untere Grenze ein festes $M<0$ her und lasse nach dem Einsetzen der Grenzen $M$ und $0$ dann [mm] $M\to-\infty$ [/mm] gehen.

Alternativ rechne das Ganze ohne Grenzen, resubstituiere und rechne alles in x aus ...

Beachte, dass auch hier die untere Grenze $x=0$ "uneigentlich" ist.

Gib dir also wieder eine feste untere Grenze [mm] $\varepsilon>0$ [/mm] vor und lasse am Ende [mm] $\varepsilon\to [/mm] 0$ gehen ...



Gruß

schachuzipus

Bezug
                        
Bezug
uneigentliche Integrale: Frage (beantwortet)
Status: (Frage) beantwortet Status 
Datum: 19:15 Mi 30.06.2010
Autor: SnafuBernd

Hi,

ich verstehe nicht ganz wieso M<0 , wo ich doch im Intervall [0,1] integriere? Eigentlich müsste es doch so auch gehen,wie ich es gemacht habe, und anschließend a->0 laufen lasse? Mein Problem war, das mein Integral nicht dem entsprach, was ich mit einem Programm rausbekommen habe.

Bezug
                                
Bezug
uneigentliche Integrale: Antwort
Status: (Antwort) fertig Status 
Datum: 07:37 Do 01.07.2010
Autor: abakus


> Hi,
>  
> ich verstehe nicht ganz wieso M<0 , wo ich doch im
> Intervall [0,1] integriere? Eigentlich müsste es doch so

Hallo,
im Interval (0,1) sind die "ln"s alle negativ.
Gruß Abakus

> auch gehen,wie ich es gemacht habe, und anschließend a->0
> laufen lasse? Mein Problem war, das mein Integral nicht dem
> entsprach, was ich mit einem Programm rausbekommen habe.


Bezug
                                
Bezug
uneigentliche Integrale: Antwort
Status: (Antwort) fertig Status 
Datum: 12:51 Do 01.07.2010
Autor: schachuzipus

Hallo nochmal,

> Hi,
>  
> ich verstehe nicht ganz wieso M<0 , wo ich doch im
> Intervall [0,1] integriere?

Mit der Substitution hast du doch das "neue" Integral in den Grenzen  [mm] $-\infty$ [/mm] bis $0$

Daher nimmm dir ne feste untere Grenze $M<0$ her und lasse am Ende [mm] $M\to-\infty$ [/mm] gehen.

> Eigentlich müsste es doch so
> auch gehen,wie ich es gemacht habe, und anschließend a->0
> laufen lasse? Mein Problem war, das mein Integral nicht dem
> entsprach, was ich mit einem Programm rausbekommen habe.

Gruß

schachuzipus

Bezug
                                        
Bezug
uneigentliche Integrale: Frage (beantwortet)
Status: (Frage) beantwortet Status 
Datum: 16:01 Do 01.07.2010
Autor: SnafuBernd

Hi,

ok, aber ist das nicht das selben als wenn ich schreibe:
[mm] \integral_{0}^{1}{ln^2(x) dx} [/mm]  sei [mm] a\in[0,1] [/mm]
=> [mm] \integral_{a}^{1}{ln^2(x) dx} [/mm]  subst.: u:=ln(x) => [mm] dx=e^u [/mm] du
[mm] =\integral_{ln(a)}^{0}{u^2e^u du} [/mm] = [mm] u^2e^u|^0_{ln(a)} [/mm] - [mm] 2ue^u|^0_{ln(a)} +2e^u|^0_{ln(a)} [/mm] =0 - [mm] ln^2(a)a [/mm] + 2ln(a)a + 2 - 2a
[mm] \limes_{a\rightarrow 0}(- ln^2(a)a [/mm] + 2ln(a)a + 2 - 2a)

hier wende ich den l'Hospital an:
[mm] \limes_{a\rightarrow 0}(- ln^2(a)a [/mm] + 2ln(a)a + 2 - 2a)  =
[mm] \limes_{a\rightarrow 0}(- \frac{ln^2(a)}{\frac{1}{a}} [/mm] + [mm] \frac{ln(a)}{\frac{1}{a}}2 [/mm] + 2 - 2a)
[mm] =\limes_{a\rightarrow 0}-\frac{2ln(a)\frac{1}{a}}{-\frac{1}{a}} [/mm] -2a +2-2a
[mm] =\limes_{a\rightarrow 0} [/mm] -2a-2a+2-2a = 2

Snafu

Bezug
                                                
Bezug
uneigentliche Integrale: Antwort
Status: (Antwort) fertig Status 
Datum: 19:19 Sa 03.07.2010
Autor: MathePower

Hallo SnafuBernd,

> Hi,
>  
> ok, aber ist das nicht das selben als wenn ich schreibe:
>  [mm]\integral_{0}^{1}{ln^2(x) dx}[/mm]  sei [mm]a\in[0,1][/mm]
> => [mm]\integral_{a}^{1}{ln^2(x) dx}[/mm]  subst.: u:=ln(x) =>
> [mm]dx=e^u[/mm] du
> [mm]=\integral_{ln(a)}^{0}{u^2e^u du}[/mm] = [mm]u^2e^u|^0_{ln(a)}[/mm] -
> [mm]2ue^u|^0_{ln(a)} +2e^u|^0_{ln(a)}[/mm] =0 - [mm]ln^2(a)a[/mm] + 2ln(a)a +
> 2 - 2a
>  [mm]\limes_{a\rightarrow 0}(- ln^2(a)a[/mm] + 2ln(a)a + 2 - 2a)
>
> hier wende ich den l'Hospital an:
>  [mm]\limes_{a\rightarrow 0}(- ln^2(a)a[/mm] + 2ln(a)a + 2 - 2a)  =
>  [mm]\limes_{a\rightarrow 0}(- \frac{ln^2(a)}{\frac{1}{a}}[/mm] +


Den Nenner [mm]\bruch{1}{a}=a^{-1}[/mm] muß Du auch ableiten.


> [mm]\frac{ln(a)}{\frac{1}{a}}2[/mm] + 2 - 2a)
> [mm]=\limes_{a\rightarrow 0}-\frac{2ln(a)\frac{1}{a}}{-\frac{1}{a}}[/mm]
> -2a +2-2a
> [mm]=\limes_{a\rightarrow 0}[/mm] -2a-2a+2-2a = 2
>  
> Snafu


Gruss
MathePower


Bezug
                                                        
Bezug
uneigentliche Integrale: Frage (beantwortet)
Status: (Frage) beantwortet Status 
Datum: 19:40 Sa 03.07.2010
Autor: SnafuBernd

Hi,

ja hab vergessen die Quadrate in denn Nenner zu schreiben, aber gerechnet habe ich mit [mm] \frac{1}{a} [/mm] => [mm] -\frac{1}{a^2}. [/mm] D.h. das Ergebnis müsste stimmen,oder?

Snafu

Bezug
                                                                
Bezug
uneigentliche Integrale: Antwort
Status: (Antwort) fertig Status 
Datum: 19:58 Sa 03.07.2010
Autor: MathePower

Hallo SnafuBernd,

> Hi,
>  
> ja hab vergessen die Quadrate in denn Nenner zu schreiben,
> aber gerechnet habe ich mit [mm]\frac{1}{a}[/mm] => [mm]-\frac{1}{a^2}.[/mm]
> D.h. das Ergebnis müsste stimmen,oder?


Das Ergebnis stimmt zwar, aber zuvor mußt Du nochmal L'Hospital anwenden.


>  
> Snafu


Gruss
MathePower

Bezug
                                                                        
Bezug
uneigentliche Integrale: Mitteilung
Status: (Mitteilung) Reaktion unnötig Status 
Datum: 20:16 Sa 03.07.2010
Autor: SnafuBernd

Hi,

ja stimmt hab da was übersprungen. Danke!

Bezug
Ansicht: [ geschachtelt ] | ^ Forum "Integration"  | ^^ Alle Foren  | ^ Forenbaum  | Materialien


^ Seitenanfang ^
ev.vorhilfe.de
[ Startseite | Mitglieder | Impressum ]